You are on page 1of 47

Professional Responsibility

I) Foundations of Professional Responsibility


a) Malpractice v. Discipline
i) Malpracticeattorneys actions were below the standard of care exercised by people
in his field.
(1) Purposecompensate plaintiff
(2) Action in a civil court
ii) Disciplineexpert testimony to prove attorney deviated from the Model Rules
(1) Not automatically malpractice but can help prove.
(2) Purposepunish lawyer
(a) Disciplinary actions are published in NY Law journal
(3) Action in a tribunal
b) Resolving Tensions in the Lawyers Role
i) Zealous Advocate lawyers are ethically and professional responsible for acting
loyally and advocating zealously on clients behalf.
ii) Officer of the Court lawyers are human beings with strongly held personal beliefs
regarding morality.
iii) Ethical Systems Influencing a Lawyer:
(1) Personal ethics
(2) Organization culture and ethics of the firm
(3) Culture and ethics of the profession itself (Model Rules)
(4) Ethics of the justice system
(5) Ethics of adversaries
(6) Ethics of the person or entity to be served (client)
c) Professionalism
i) Definitions:
(1) Certain behaviors that are, at a minimum, to be expected of people of the
occupation.
(2) Special skills are required because clients have no idea what they are doing.
(3) Special expertise and ethical responsibilities. ABA Commission on
Professionalism
(4) An occupation whose members have special privileges, such as exclusive
licensing [monopoly] justified by these assumptions, that:
(a) Practice requires substantial intellectual training and use of complex
judgments;
(b) Client trust based on inability to evaluate adequately the quality of service;
(c) Clients trust presupposes that the practitioners self-interest is overbalanced
by devotion to serving both clients interest and public good; and
(d) That the occupation if self-regulating.
ii) NY CPLR 214(6)3 year statute of limitations for malpractice case against
professionals, but in medical malpractice limit is 2 years. Breach of contract is 6
years.
1

(1) Insurance agents and brokers dont count.


d) In re Paulter (2002)
i) Facts: Neal refused to turn himself in for murder unless provided a public defender.
Deputy District Attorney Paulter pretended to be a public defender. He advised Neal
to turn himself in. Neal listened because he believed he was being represented by an
attorney. The Court finally appointed Neal a real public defender but Neal fired him
and got the death penalty.
ii) Paulter tried to argue that there was an imminent public harm. He believed his
actions were justified to apprehend a manic serial killer. He also argued that he never
provided legal advice to Neal.
iii) Colorado Supreme Court held that his actions are not justified because there was not
an imminent public harm.
(1) They said even a noble motive does not warrant a departure from the Rules.
(2) Paulter was suspended for 3 months and received 12 months probation.
iv) Takeaway=follow the rules of professional conduct.

II) Regulating Attorney Conduct


a) History of Model Rules
i) 1908 ABA Canons of Ethicsfirst comprehensive attempt by the legal profession to
regulate itself
(1) Deliberately vague because drafters were worried that no code could particularize
all the situations lawyers would face.
(2) Enforcement was intermittent, haphazard, and often biased against solo and small
firm practitioners.
ii) 1969 Model Code of Professional Responsibility
(1) Three Sections:
(a) Canonsdescribed general professional conduct
2

(b) Ethical considerationsaspirational


(c) Disciplinary rulesset a floor for professional conduct
(2) Weaknesses:
(a) Too concerned with trial lawyers and not transactional/negotiators
(b) Didnt take into account large firm, multijurisdictional practice
(c) No provision regarding subsequent representation adverse to interests of
former client
iii) 1983 ABA Model Rules of Professional Conduct
(1) Black letter rules with no explanatory comments.
(a) Mandatorymust do; require disciplinary action when violated
(b) Permissiveallowed to do; no disciplinary action when violated (just a guide)
(c) Aspirationalspeak to level of conduct lawyer should strive to achieve
(d) Commentsdo not add obligations
(2) Model Rules have been adopted by 49 states.
(a) NY became the 48th state to adopt the Model Rules (April 1, 2009).
(b) California is the only state not to adopt the Model Rules.
(3) Model Rules designed to be client oriented.
b) Philosophy of Lawyering
i) Lawyer as Agent
(1) Express and implied authority to use skill on behalf of client.
(a) Express by an actual agreement
(b) Implied result of attorney client relationship.
(2) Acting for client means lawyers conduct will be attributable to the client; both
good and bad.
(a) Binding the clientsigning contracts on behalf of a client.
(i) SEC v. McNultyin litigation, attorneys default, mistake or omission can
be attributed to client and client may not be relieved.
(3) A litigant chooses counsel at his peril.
ii) Lawyer as Fiduciary
(1) Must place clients interests above their own.
(a) Unique position of trust and confidence.
(2) Reasons for highest level of fiduciary duty:
(a) Client depends on lawyers superior knowledge/judgment, integrity and
fairness and put aside caution that is customary when dealing with others on
important matters.
(b) Lawyers get information about client that gives an unfair advantage in
dealings between them.
(c) Client may be financially or psychologically dependent on attorneys
continued representationnot so easy to change attorneys mid-way.
(3) Benson v. State Barattorney borrowed money from client in return for IOUs
(a) Court disbarred attorneyabused clients trust
(4) In re Lerner (NY)lawyer recommends firm because hed get a cut of the money
(a) A fiduciary may be required to disgorge any ill-gotten gain even where the
plaintiff has sustained no direct economic loss.
3

iii) Loyalty and Diligence


(1) Lawyer must pursue clients objectives without being conflicted by other
responsibilities or interests.
(a) Survives termination of attorney-client relationshiplawyer cannot act
adversely to a former client in a substantially related matter.
(2) Duties
(a) Pursue clients interests without undue delay.
(b) Undivided loyaltiesundermine ability to be diligent and threatens fiduciary
position.
(3) Entire devotion to clients interest, zeal, exertion of utmost learning and ability.
c) Admission to Practice
i) Authority to control Admission
(1) StateSupreme Court governs rules and procedures to be admitted to practice
(a) Licensing of attorneys in a state is in exclusive control of the highest court in
the state.
(i) ExceptionNY the appellate departments govern bar admission
(b) Source of power
(i) This is in the definition of what it means to be a courtpower vests upon
creation of court; inherent power.
(ii) NJ and PA are two states with explicit grants of power.
(2) Federal
(a) Admission to state court DOES NOT automatically entitle practice in federal
court.
(i) Must also be admitted to the federal bar.
(b) Qualifications:
(i) Must be a lawyer in the state where the district court sits.
(ii) Supreme Court and other jurisdictions allow admission based on prior
admission to another federal court.
(iii)
Need to be introduced to court by a current member of the federal
bar.
(3) Pro hac vice
(a) For this turn only you can be admitted to practice before a court that you are
not a member of.
(b) Must submit a motion to the court
(i) The court will likely require some kind of association with a local firm
because an attorney needs to be subject to discipline of the court.
(c) This is at the courts discretion.
(i) Criminal defendants cannot claim right to counsel if their attorney of
choice is not admitted pro hac vice.
(d) There is a limit to the number of times you can be admitted pro hac vice.
(i) Not an explicit number but it will be denied if you are trying to circumvent
state admission procedures.
ii) Good Moral Character
(1) Applicant has burden of proving good moral character, it is not assumed.
4

(2) A lawyer representing a client/applicant for bar admission is governed by rule


applicable to client/lawyer relationship (MR 1.6).
(a) Duty to client superseded by reporting requirements of MR 8.1
(3) Duty to disclose:
(a) Must disclose convictions.
(b) Must disclose arrest IF askedcannot refuse to answer on 1st Amendment
grounds.
(i) Failure to disclose an arrest, even as a juvenile, can be an independent
basis for denying admission, even if arrest itself is not a basis for denial.
(c) Do not need to disclose parking tickets unless application asks elsewhere.
(i) Failure to pay parking tickets is evidence of fiscal
irresponsibility/disrespect for the law.
iii) Morality
(1) Courts can deny admission if they view applicants activities, even if legal, as
engaging in unauthorized practice of law or having disrespect for the legal
system.
iv) Financial habits
(1) Poor financial patterns can be a basis for denial until steps/efforts are made to
repay debts.
v) Wieder v. Skala (NY Court of Appeals)
(1) Is it a public policy violation when a law firm fires a lawyer for reporting
disciplinary violation by another attorney?
(a) Yesthis case create a new cause of action available only to lawyers.
(i) Wrongful termination based on a law firm requiring an employee to
violate professional ethics rule.
1. Applied very narrowlyonly when lawyer would be required to report
another lawyer and firm threatens to fire such lawyer.
Model Rule 8.1Bar Admission and Disciplinary Matters
An applicant for admission to the bar, or a lawyer in connection with a bar admission application or in
connection with a disciplinary matter, shall not:
(a) knowingly make a false statement of material fact; or
(b) fail to disclose a fact necessary to correct a misapprehension known by the person to have
arisen in the matter, or knowingly fail to respond to a lawful demand for information from an
admissions or disciplinary authority, except that this rule does not require disclosure of
information otherwise protected by Rule 1.6.
Model Rule 5.2Responsibilities of a Subordinate Lawyer
(a) A lawyer is bound by the Rules of Professional conduct notwithstanding that the lawyer acted at the
direction of another person.
(b) A subordinate lawyer does not violate the Rules of Professional Conduct if that lawyer acts in
accordance with a supervisory lawyers reasonable resolution of an arguable question of professional duty.
Model Rule 8.3Reporting Professional Misconduct
(a) A lawyer who knows that another lawyer has committed a violation of the Rules of Professional Conduct
that raises a substantial question as to that lawyers honesty, trustworthiness or fitness as a lawyer in other
respects, shall inform the appropriate professional authority.
(b) A lawyer who knows that a judge has committed a violation of applicable rules of judicial conduct that
raises a substantial question as to the judges fitness for office shall inform the appropriate authority.
(c) This Rule does not require disclosure of information otherwise protected by rule 1.6 or information
gained by a lawyer or judge while participating in an approved lawyers assistance program.

Model Rule 1.0Terminology


(f) Knowingly, known, or knows denotes actual knowledge of the fact in question. A persons
knowledge may be inferred from circumstances.

III) Criminal CasesClient-Lawyer Relationship


a) Competency
i) Determining skill required:
(1) Complexity and specialized nature of matter
(2) General experience
(3) Training and experience in the field in question
(a) Specialists are held to a higher standard if they advertise such specialty
(4) Preparation and study lawyer is able to give to the matter
(5) Whether it is feasible to refer the matter to, or consult with, a lawyer of
established competence in the field
ii) Enforcing duty of competence
(1) Disciplinary action
(a) Rare unless egregious conduct
(b) Special circumstances are irrelevant (e.g., personal crises)
(i) Can be a mitigating factor but still subject to sanctions
(2) Malpractice action
(a) Intentional tort, break of fiduciary duty, breach of K
(b) Negligence
(i) Duty of care to clients
(ii) Breach of duty of carecompetence normally exercised by attorneys in
similar circumstances is the standard
1. Custom is dispositive
2. Expert testimony required to define the standard
(iii)
But for cause of actionprove that you would have won the
case
(iv)Damagesvalue of underlying action
(v) Violation of a disciplinary action is evidence of negligence that the trier of
fact can consider.
(3) Ineffective assistance of counsel (6th Amendment)
6

(a)
(b)
(c)
(d)

Only criminal cases


Lawyer is not a party to the action
Must be a causal relationship between professional failure and injury to client
Strickland v. WA
(i) Defendant must show:
1. Lawyers conduct fell below range of professional acceptance; AND
a. Courts give great deference to tactical decisions of attorneys in
these cases.
2. Client suffered prejudice in the sense that verdict was probably
affected by lawyers incompetence.
a. Court presumes prejudice in certain situations:
i. Conflict of interest
ii. Defense counsel fails to follow up on Ds instruction to appeal
iii. Proceedings could not reasonably be considered adversarial
(e) Remedynew trial
(4) Sanctions (least to most severe)
(a) Warning
(b) Private reprimand/censure
(c) Requirement of CLE classes
(d) Community service
(e) Public reprimand/censure (published in law journal)
(f) Monetary fines
(g) Interim suspensionlawyer suspended immediately because of danger posed
to client (e.g., alcohol abuse)
(h) Full suspensioncan reapply
(i) Disbarmenthave to be readmitted to the bar
Model Rule 1.1Competence
A lawyer shall provide competent representation to a client. Competent representation requires the legal knowledge,
skill thoroughness and preparation reasonably necessary for the representation.

b) Fees
i) Determining the Fee:
(1) When you take on a client, you agree not to take on any other business that will
take away from duty to diligently represent that client.
(a) You give up the right to take on excessive business and the right to take on
business that might conflict with your clients business.
Model Rule 1.5Fees
(a) A lawyer shall not make an agreement for, charge or collect an unreasonable fee or an unreasonable amount for
expenses. The factors to be considered in determining the reasonableness of a fee include the following:
(1) the time and labor required, the novelty and difficulty of the questions involved, and the skill requisite to
perform the legal service properly;
(2) the likelihood, if apparent to the client, that the acceptance of the particular employment will preclude other
employment by the lawyer;
(3) the fee customarily charged in the locality for similar legal services;
(4) the amount involved and the results obtained;
(5) the time limitations imposed by the client or by the circumstances;
(6) the nature and length of the professional relationship with the client;
(7) the experience, reputation, and ability of the7 lawyer or lawyers performing the services; and
(8) whether the fee is fixed or contingent.

Model Rule 1.5Fees


(b) The scope of the representation and the basis or rate of the fee and expenses for which the client will be
responsible shall be communicated to the client, preferably in writing, before or within a reasonable time after
commencing the representation, except when the lawyer will charge a regularly represented client on the same basis
or rate. Any changes in the basis or rate of the fee or expenses shall also be communicated to the client.
(e) A division of a fee between lawyers who are not in the same firm may be made only if:
(1) the division is in proportion to the services performed by each lawyer or lawyer assumes joint responsibility
for the representation;
(2) the client agrees to the arrangement, including the share each lawyer will receiver, and the agreement is
confirmed in writing; and
(3) the total fee is reasonable.

ii) Types of Fees:


(1) Flat Fee lawyer charges to complete a given service, regardless of the time it
takes; must be reasonable and partially refundable; creates incentive to be
efficient
(a) Saves the lawyer time by not calculating hours.
(b) Low flat fees attract clients and your volume of clients will increase.
(c) If client terminates before job is complete the lawyer must refund part of the
fee and charge a percentage based on the work done or an hourly fee.
(i) Retainer should state an hourly price in case of cancellation.
(2) Hourly Fee charge for each hour you work, attorney gets paid win or lose
(a) Produces inefficient work and encourages unnecessary projects.
(b) Decreased associate satisfaction.
(c) Lack of mentoringif youre mentoring youre not billing.
(d) Decreased Pro Bono work.
(3) Contingency Fee no fee unless client obtains a monetary recovery
(a) Encourages nuisance lawsuits that clog the court.
(b) Unjustly enrich lawyers who charge substantial contingent fees.
(c) Encourages settlement when there shouldnt be.
(d) Prohibits in matrimonial and criminal cases because. . .
(i) A plea may not be in the best interest of the client, and
(ii) We do not want to encourage divorce.
(e) Must be in writing and signed by the client.
(f) Reverse contingencytake part of what lawyer saved the client
(4) Value Billing charge based on value of work for client
8

(a) Lawyer and client negotiate value of work at the beginning (similar to flat
fee).
(b) Lawyer and client negotiate at the end; if the negotiation fails then a formula
is used (similar to contingent fee).
(c) Lawyer unilaterally determines the value at the end of the matter.
(d) Used by courts when awarding fees under fee-shifting statutes.
(e) Usually used in civil rights and environmental cases (value of the win is high
but the lawyer invests endless hours); its a win for a society.
(5) Hybrid Fee combines two types
(a) E.g., contingent hourly ratehourly rate only paid if client wins
(6) Retainers
(a) General retainerfee to make legal services available when needed during
specified time
(i) Fee is earned when paid because attorney is entitled to the money
regardless of whether he actually performs any services.
(b) Special retainer payments of funds for a specific service, fees not earned
until actual work performed; money paid into trust fund
(i) Fee must be refundable.
1. Non-refundable special retainers are void as unethical.
(ii) Must be placed in a trust if advanced payment
(iii)
There is a presumption that a special retainer exists, an attorney
must show by a convincing preponderance of the evidence that it is
something else.
(iv)Iowa Supreme Court v. Aplandflat fee paid in cash, deposited into
portfolio, attorney agreed to pay all expenses out of $5K paid; no written
agreement but oral agreement said it would include appeals; 6 months
after case ended, client demanded a portion of money back but lawyer said
he paid for depositions out of money. Attorney returned $2,000 but clients
filed grievance.
1. Court drew distinction between general retainers, special retainers and
flat fees.
2. There were six ethical violations by Aplandunrefundable retainer,
didnt deposit into trust, failed to respond to disciplinary charges,
didnt give an accounting of time spent, and tried to pay them to
withdraw the complaint.

Model Rule 1.5Fees


(c) A fee may be contingent on the outcome of the matter for which the service is rendered, except in a matter in
which a contingent fee is prohibited by paragraph (d) or other law. A contingent fee agreement shall be in a writing
signed by the client and shall state the method by which the fee is to be determined, including the percentage or
percentages that shall accrue to the lawyer in the event of settlement, trial, or appeal; litigation and other expenses to
be deducted from the recovery; and whether such expenses are to be deducted before or after the contingent fee is
calculated. The agreement must clearly notify the client of any expenses for which the client will be liable whether
or not the client is the prevailing party. Upon conclusion of a contingent fee matter, the lawyer shall provide the
client with a written statement stating the outcome of the matter and, if there is a recovery, showing the remittance to
the client and the method of its determination.
(d) A lawyer shall not enter into an arrangement for, charge, or collect:
(1) any fee in a domestic relations matter, the payment or amount of which is contingent upon the securing
of a divorce or upon the amount of alimony or support, or property settlement in lieu thereof; or
(2) a contingent fee for representing a defendant in a criminal case.
Model Rule 1.8Conflict of Interest: Current Clients: Specific Rules
(d) Prior to the conclusion of representation of a client, a lawyer shall not make or negotiate an agreement giving the
lawyer literary or media rights to a portrayal or account based in substantial part on information relating to the
representation.
Model Rule 1.15Safekeeping Property
(a) A lawyer shall hold property of clients or third persons that is in a lawyers possession in connection with a
representation separate from the lawyers own property. Funds shall be kept in a separate account maintained in the
state where the lawyers office is situated, or elsewhere with the consent of the client or third person. Other property
shall be identified as such and appropriately safeguarded. Complete records of such accounts funds and other
property shall be kept by the lawyer and shall be preserved for a period of five years after termination of the
representation.
(b) A lawyer may deposit the lawyers own funds in a client trust account for the sole purpose of paying bank service
charges on that account, but only in an amount necessary for that purpose.
(c) A lawyer shall deposit into a client trust account legal fees and expenses that have been paid in advance, to be
withdrawn by the lawyer only as fees are earned or expenses incurred.
(d) Upon receiving funds or other property in which a client or third person has an interest, a lawyer shall promptly
notify the client of third person. Except as stated in this rule or otherwise permitted by law or by agreement with the
client, a lawyer shall promptly deliver to the client or third person any funds or other property that the client or third
person is entitled to receive and, upon request by the client or third person, shall promptly render a full accounting
regarding such property.
(e) When in the course of representation a lawyer is in possession of property in which two or more persons (one of
whom may be the lawyer) claims interests, the property shall be kept separate by the lawyer until the dispute is
resolved. The lawyer shall promptly distribute all portions of the property as to which the interests are not in dispute.

IV) Criminal CasesConfidentiality


a) Secrecy/Confidentiality
i) What is Secrecy?
(1) Lawyers ethical obligation not to use or reveal information relating to
representation of a client.
10

(a) Information may come from a client, documents, third parties or attorneys
own observations.
(2) Rules of Ethics=Duty of Confidentiality
(3) Rules of Evidence=Attorney Client Privilege
(4) Rules of Civil Procedure=Work Product Doctrine
ii) Who is a client?
(1) Does this person believe she is your client?
(a) If so, is that reasonable in light of all the circumstances?
(i) Factors considered:
1. Expectation of confidentiality
2. Speaking to you in your capacity as a lawyer
3. Seeking legal advice
(ii) What is unreasonable?
1. Someone just looking at your website without any direct
communication.
(b) Everything is look at from the clients perspective.
(i) You must be clear and explicit about when representation begins and ends.
(ii) Must keep confidential information received from prospective client
confidential and must decline representation of someone else if that
confidential information would materially limit your ability to represent
new person.
1. If there is proof that prospective client was not seeking legal
representation but just trying to create future conflicts of interest, then
no fiduciary relationship arises and no duty of confidentiality.
b) Three categories of information:
i) Duty of confidentiality prohibits lawyer from voluntarily using or revealing
information that they know about a client or clients matter, regardless of source
(relating to representation)
(1) Under MR 1.6 you have a professional duty not to reveal to others any
information relating to the representation.
(a) Scope of rule is enormousprohibited from disclosing anything at all relating
to representation.
Model Rule 1.6(a)Confidentiality of Information
A lawyer shall not reveal information relating to the representation of a client unless the client gives
informed consent, the disclosure is impliedly authorized in order to carry out the representation or the
disclosure is permitted by paragraph (b).

ii) Attorney-Client Privilege prevents anyone from compelling disclosure of


communications between an attorney and client when the client is seeking legal
advice; anything protected by this is protected by MR 1.6; stronger but narrower
scope than duty of confidentiality
11

(1) Applies only when the five Cs are present:


(a) Client
(i) If you agree to accept someones civil case but not criminal, everything
you talked about is protected because they were a prospective client.
(b) Communicate
(i) Oral, written and email (even if not marked confidential).
1. Hand gestures count too.
(c) Confidentially with
(i) Only in presence of client and lawyer unless someone is there to assist in
representation OR family member.
(d) Counsel (lawyer acting as lawyer)
(i) Includes conversation/phone message with secretary (agent).
(e) To obtain Counsel (Legal advice)
(2) Facts v. Communications
(a) Facts are not protected.
(i) E.g., what color the light was?
(b) Communications are protected.
(3) It can be waived but it is all or nothing, cannot waive part of the privilege.
(a) Intentionally waiving the privilege.
(i) Turning over confidential documents.
(ii) Client reveals something during deposition.
(b) Waiver can be accidental.
(c) Implied authority that attorney can reveal confidential information that will
advance a settlement.
(4) Exceptions: (JACSS)
(a) Joint clientstwo clients represented by a single attorney
(i) If dispute arises, attorney-client privilege is broken between all parties.
(ii) Whatever conversation you had between each client is no longer
privileged and anyone can inquire into communication.
(b) Advice of Counsel Defense
(i) If client says they did something not knowing it was illegal because
attorney said it was ok the privileged is waived.
(c) Crime-frauddepends on clients intent, not attorneys knowledge
(i)
(d) Self defense
(i) If client sues lawyer.
(e) Subpoena
(i) Object to subpoena until the court orders you to produce it.
1. If judge orders you to produce it, you can produce without violating
ethical duties because there is a court order.
(5) Entity Clients
12

(a) Attorney who represents corporation can claim attorney-client privilege for
communications with some officers, directors and employees (always
President and CEO).
(b) Control Group Test
(i) Westinghousecorporations are protected only if person communicating
with lawyer is a member of a control groupelite group of corporate
officers and employees who actually control corporation and make its
policies.
1. This test is abandoned in most jurisdictions because it is too narrow.
(c) Subject Matter Test
(i) Upjohnconversations between corporate attorney and employee, even if
initiated by attorney, are privileged as long as conversations are for the
purpose of gathering information necessary to give or implement legal
advice to corporation.
1. Focuses on nature of communication, not status of communicator.
2. Criticized for being too broad.
iii) Work Product Doctrine protects information created by lawyer for purposes of
litigation (FRCP 26)
(1) Prevents discovery of materials prepared in anticipation of litigation.
(a) Must be tangible (not communications).
(b) Includes phone messages taken by secretary.
(c) Lawyer cannot be compelled to reveal written material prepared in
anticipation of litigation.
(d) Includes letters written from client to lawyer.
(e) Lawyer runs risks of waving by showing certain documents to a witness while
on the stand (memos etc.)
(2) Hickman v. Taylor
(a) Ordinary Work Productinformation recorded by the attorney because of
litigation.
(i) FRE 26(b)(3)two part test of discovery of work product:
1. Substantial need of materials; AND
2. Unable to obtain without undue hardship the materials or
substantial equivalent by other means.
(b) Opinion work productwritten impressions, conclusions, opinion or legal
theories of an attorney concerning litigation
(i) Virtually immune from discovery, but subject to waiver.
(3) Inadvertent disclosure
(a) Attorney-client privilege is not waived if attorney takes reasonable steps to
make sure this doesnt happen.
(i) If no reasonable steps, court can conclude ACP is waived.
Model Rule 4.4Repsect for Rights of Third Persons
(a) In representing a client, a lawyer shall not use means that have no substantial purpose other than to
13 or use methods of obtaining evidence that violate the legal
embarrass, delay, or burden a third person,
rights of such a person.
(b) A lawyer who receives a document relating to the representation of the lawyers client and knows or
reasonable should know that the document was inadvertently sent shall promptly notify the sender.

c) Exceptions to the Duty of Confidentiality


i) Thirteen exceptions (#1-9 are derived from MR 1.6) (#10-13 are found in MR 1.9,
1.13, 3.3(a) and 3.3(b)).
(1) Informed ConsentMR 1.6(a)
(a) Must tell what you are going to disclose, facts giving rise to situation, material
advantages and disadvantages, and alternatives.
MR 1.0Terminology
(e) Informed consent denotes the agreement by a person to a proposed course of conduct after
the lawyer has communicated adequate information and explanation about the material risks of
and reasonably available alternatives to the proposed course of conduct.

(2) Implied AuthorityMR 1.6(a)


(a) The disclosures are required by the rules of procedures; or
(b) The disclosures will help the client, not hurt the client.
(i) Never implied authority to hurt client.
(ii) Never implied authority to override clients instructions not to disclose.
(3) To prevent reasonably certain death or substantial bodily harmMR 1.6(b)(1)
(a) Does not have to be the act of the client
(i) E.g., could be a witness plotting to blow up school and has done so in past.
Can reveal a future plan, but not past because harm already happened.
(b) Does not have to be a criminal act
(c) Does not have to be imminent
(d) Disclosure can be no broader than necessary to prevent future harm
(e) Disclosure is not mandatory
(4) To prevent the client from committing a crime or fraud that is reasonable certain
to result in substantial injury to the financial interests or property of another
and in furtherance of which the client has used or is using the lawyers services
MR 1.6(b)(2)
(a) Applies only when the client is using (or has used) the lawyers services to
commit a future crime or fraud that will almost certainly harm someone elses
property substantially.
(5) To prevent, mitigate or rectify substantial injury to the financial interests or
property of another that is reasonably certain to result or has resulted from the
clients commission of a crime or fraud in furtherance of which the client has
used the lawyers servicesMR 1.6(b)(3)
(a) Applies after crime has been committed to prevent future damage.
(i) Prior acts from which there will be subsequent damage.
(6) To secure legal advice about the lawyers compliance with these rulesMR
1.6(b)(4)
(a) Seek guidance about whether something you are doing is compliant with
rules.
14

(i) Disclosing information to secure such advice is usually impliedly


authorized for a lawyer to carry out representation.
(7) Lawyers claim or self-defenseMR 1.6(b)(5)
(a) Permits disclosure to the extent the lawyer reasonably believes necessary in
order to:
(i) Establish a claim or defense on behalf of the lawyer and the client (e.g.,
free dispute or legal malpractice claim)
(ii) Establish a defense to a criminal charge or civil claim against the lawyer
based upon conduct in which the client was involved
(iii)
Respond to allegations in any proceeding concerning the lawyers
representation of the client.
(8) To comply with some other lawMR 1.6(b)(6)
(a) Attorney must confer with client and tell about disclosure and the effects.
(9) Court orderMR 1.6(b)(6)
(a) Discretionary but can be subjection to contempt of court.
(10)
Information that became generally known after a representation ended
MR 1.9(c)
(11)
False evidence offered to a tribunalMR 3.3(a)(3)
(a) Attorney is required to:
(i) Notify court that perjury has taken place and remedy the situation.
(12)
Criminal or fraudulent conduct related to proceedings before a tribunal
MR 3.3(b)
(a) Required disclosure
(i) E.g., witness tampering
(13)
When the client is an organization, a clear violation of law may be
revealed to prevent substantial injury to the organizationMR 1.13
(a) Required to report internally within the organization.
(i) If this does not rectify situation, lawyer may report out of organization.
(b) Willful blindness applies, once the attorney realizes there is wrongdoing, duty
to report up the chain of command.
MR 1.6Confidentiality of Information
(a) A lawyer shall not reveal information relating to the representation of a client unless the client gives informed
consent, the disclosure is impliedly authorized in order to carry out the representation or the disclosure is permitted
by paragraph (b).
(b) A lawyer may reveal information relating to the representation of a client to the extent the lawyer reasonable
believes necessary:
(1) to prevent reasonably certain death or substantial bodily harm;
(2) to prevent the client from committing a crime or fraud that is reasonably certain to result in substantial
injury to the financial interests or property of another and in furtherance of which the client has used or is
using the lawyers services;
(3) to prevent, mitigate or rectify substantial injury to the financial interests or property of another that is
reasonably certain to result or has resulted from the clients commission of a crime or fraud in furtherance
of which the client has used the lawyers services;
(4) to secure legal advice about the lawyers compliance with these Rules;
(5) to establish a claim or defense on behalf of the lawyer in a controversy between the lawyer and the
client, to establish a defense to a criminal charge or civil claim against the lawyer based upon conduct in
which the client was involved, or to respond to allegation in any proceeding concerning the lawyers
representation of the client; or
(6) to comply with other law or a court order.

15

d) Attorney Client Privilege and Physical Evidence


i) Types of Physical Evidence:
(1) Given to you by a client
(2) Given to you by a third party
(3) You find on your own
(4) Evidence that you only see, but do not touch
(a) E.g., client gives you tour of crime scene
(5) You only hear about, but do not see or touch
(a) E.g., someone tells you there is evidence but you dont go find it
ii) Handling Physical Evidence
(1) In re Ryder
(a) Ryders client told him there was evidence (shotgun and money) in a safe
deposit box and Ryder took the evidence and moved it into his own box so he
can make an argument to suppress on grounds of attorney-client privilege.
(b) Court held that Ryders conduct was not encompassed by attorney-client
privilege and that Ryder was an active participant in the crime.
(c) What is covered by attorney-client privilege?
(i) Anything defendant told Ryder.
(d) What is not covered?
(i) Ryder collecting and hiding the evidence.
(ii) Once he touches the evidence it is no longer covered by the privilege.
(2) People v. Meredith
(a) An observation by defense counsel or his investigator, which is the product of
privileged communication (e.g., D tells you where the wallet is hidden) cannot
be admitted as evidence at trial UNLESS defense, by altering or removing
physical evidence, has precluded the prosecution from making the same
observation.
(i) If you pick it up, you have to disclose because your fingerprints are on it
so you altered the evidence.
(3) Once you get your hands on it, you are liable!
16

iii) Delivering Evidence to the Prosecution


(1) Defense lawyers in a criminal case are prohibited from hiding physical evidence
in their offices.
(2) If a defense lawyer actually takes possession of physical evidence, they must turn
it over to the prosecution.
(a) Criminal defense lawyer must give authorities physical evidence that lawyer
receivers from client if evidence is material to the case.
(3) Courts have difficulty in determining exactly what the prosecution can say to the
jury about the source of the evidence.
iv) Destruction of Evidence
(1) In most states, destroying physical evidence is a crime even before subpoena is
issued.
(2) Under federal law, intentional destruction after subpoena is an obstruction of
justice.
(a) Destruction of evidence before a subpoena is issued is a federal crime only if:
(i) Evidence is relevant to a pending grand jury or criminal investigation; and
(ii) Destruction was done with corrupt or evil intent.
(3) The least detrimental way to turn over evidence is:
(a) Send it in anonymous; or
(b) Hire attorney to represent lawyer and have them bring it to authorities and
have them invoke attorney-client privilege about who your client is.
(4) Intentional destruction of a document to prevent its use at trial, even when not
illegal, creates an adverse inference that the partys entire case is weak.
(a) A good lawyer can almost always explain a damaging document to the jury
but a missing one creates problems because a jury will make adverse
inferences.
MR 3.4Fairness to Opposing Party and Counsel
A lawyer shall not:
(a) unlawfully obstruct another partys access to evidence or unlawfully alter, destroy or conceal a
document or other material having potential evidentiary value.

v) ABA Standard for Criminal justiceStandard 4-4.6


(a) Defense counsel who receives physical item under circumstances implicating a
client in criminal conduct should disclose location or deliver to law enforcement
only:
(i) Required by law or court order; or
(ii) As provided in (d)
(b) Unless required to disclose, defense counsel shall return item to source from
whom it was received, except (c) and (d). In returning the item to the source,
defense counsel should advise the source of the legal consequences pertaining to
possession or destruction of the item. Defense counsel should also prepare a

17

written record of these events for his or her file, but should not give the source a
copy of such record.
(c) Defense counsel may receive the item for a reasonable period of time during
which defense counsel:
(i) Intends to return it to the owner;
(ii) Reasonably fears that return of the item to the source will result in
destruction of the item;
(iii) Reasonable fears return will result in physical harm;
(iv) Intends to test, examine, inspect, or use the item in any way as part of
defense counsels representation of the client; or
(v) Cannot return it to the source.
(d) If the item receive is contraband, defense may suggest client destroy it if no
pending case or investigation and not clearly in violation of criminal statute. If
such destruction is not permitted by law and defense counsel does not think he
can reasonable retain the item, whether or not contraband, in a way that does not
pose unreasonable risk to anyone, defense counsel should disclose location or
deliver item to law enforcement.
(e) If defense counsel disclosed the location of or delivers the item to law
enforcement authorities, he should do so in a way best designed to protect the
clients interests.
vi) Problem of perjury (What if your client liesor plans to lieat trial?)
(1) Nix v. Whiteside
(a) Defendant stabbed a drug dealer, claiming self defense. Defendant told his
lawyer that he never actually saw a gun but thought the dealer was reaching
for one.
(b) A week before trial, the defendant said If I dont say I saw a gun, Im dead.
(i) His attorney would not let him testify and threatened to withdraw or tell
the court if he testified falsely.
(c) Defendant ended up testifying that he knew the drug dealer owned a gun and
thought he was reaching for one but not that he actually saw one.
(i) Defendant convicted and claimed ineffective assistance of counsel.
(d) Supreme Court held that attorneys conduct did not amount to ineffective
assistance of counsel.
(i) The right to testify does not include the right to testify falsely.
1. The used the Strickland test: (1) serious attorney error below the range
of acceptable conduct; and (2) prejudice.
vii) Prospective v. Retrospective Perjury
(1) Prospective
(a) If a lawyer knows that a criminal defendant will testify falsely, the lawyer
must not put that into evidence.
18

(i) If a lawyer reasonably believes that a criminal defendant will testify


falsely (but the lawyer does not know), then the lawyer must put client on
the stand if the client decides to testify.
(ii) In a civil case, if a lawyer reasonably believes that a client or any other
witness will testify falsely, then the lawyer has discretion whether or not to
call the person to the stand.
(b) Strategic lawyeringnever tell client that they have to comply with the law,
try to figure out a way to deal with a situation and expose your client to the
least amount of negative effects.
(c) MR 3.3 is the only rule that expressly overrides the duty of confidentiality
under MR 1.6
(2) Retrospective
(a) If a lawyer knows of falsity, must take reasonable remedial measures, such as:
(i) Keep client off the stand
(ii) Ask only questions that will not elicit false testimonynarrative.
(iii)
Move to withdraw.
(iv)Four Rs (1) Recess proceeding; (2) Remonstrate with the client;
(3) Resign (withdraw); and (4) Reveal if only effective option.
(b) Must balance the trilemma of the duty of confidentiality, duty of competence
and duty of candor with the tribunal.
(i) All three will not mesh when you know a client will lie on the stand.
MR 2.1Advisor
In representing a client, a lawyer shall exercise independent professional judgment and render candid
advice. In rendering advice, a lawyer may refer not only to law but to other considerations such as moral,
economic, social and political factors that may be relevant to the clients situation.
MR. 3.3Candow Toward the Tribunal
(a) A lawyer shall not knowingly:
(1) make a false statement of fact or law to a tribunal or fail to correct a false statement of material fact
or law previously made to the tribunal by the lawyer;
(2) fail to disclose to the tribunal legal authority in the controlling jurisdiction known to the lawyer to
be directly adverse to the position of the client and not disclosed by opposing counsel; or
(3) offer evidence that the lawyer knows to be false. If a lawyer, the lawyers client, or a witness called
by the lawyer, has offered material evidence and the lawyer comes to know of its falsity, the lawyer
shall take reasonable remedial measures, including, if necessary, disclosure to the tribunal. A lawyer
may refuse to offer evidence, other than the testimony of a defendant in a criminal matter, that the
lawyer reasonably believes if false.
(b) A lawyer who represents a client in an adjudicative proceeding and who knows that a person intends
to engage, is engaging or has engaged in criminal or fraudulent conduct related to the proceeding shall
take reasonable remedial measures, including, if necessary, disclose to the tribunal.
(c) The duties stated in paragraphs (A) and (b) continue to the conclusion of the proceeding, and apply
even if compliance requires disclosure of information otherwise protected by Rule 1.6.
(d) In an ex parte proceeding, a lawyer shall inform the tribunal of all material facts known to the lawyer
that will enable the tribunal to make an informed decision, whether or not the facts are adverse.

19

V)Criminal CasesConflicts of Interest


*Enforcement mechanism is a motion to disqualify.
a) Costscan cost lawyers a lot of money
i) Erroneously accepting a matter can cost money.
ii) Erroneously rejecting a matter costs money.
iii) Erroneously continuing a matter costs money.
b) Foundation
i) Secrecy
ii) Loyaltytotal devotion to the interests of a client [do nothing to harm or distract
your pursuing such]
(1) Duty of loyalty is compromised if a significant risk that lawyers ability to
consider, recommend, or carry out an appropriate course of action for client will
be materially limited.
(a) E.g., lawyer is tempted to give up an option (defense, cause of action,
investigations) because interests are directed elsewhere.
(2) Most courts refuse to allow hot potato scenariowithdraw from representation
of current client to represent another, better client to avoid conflict of interest.
c) Concurrent Conflicts
i) Direct Adversity Conflictswhenever a lawyer directly opposes a current clients
(1) Loyalty to a current client prohibits undertaking representation directly adverse
to that client without that clients informed consent. (under very limited
circumstances)
ii) Materially Limiting Conflictswhenever a lawyers loyalty to a current client is or
may be materially compromised by a competing loyalty to any other person (whether
a current client, former client, or some other third person), or by the lawyers own
personal interests
(1) Conflict exists if there is a significant risk that a lawyers ability to consider,
recommend or carry out an appropriate course of action for the client will be
materially limited as a result of the lawyers other responsibilities or interests.
(2) Arise far more often than direct adversity conflicts.
iii) Competing Interests
(1) Another client (meaning another current client)
(2) A former client
(3) Some other third person (such as a friend, relative, or spouse) who is neither a
client nor former client
(4) The lawyers own persona, financial, political, or social interests
d) Who is a client? Someone who reasonably believes he is a current client.
i) Current Clients
20

(1) Anyone who reasonably believes you are their lawyer and is reasonably relying
on you to perform legal services. May consider themselves such at any of 4
stages:
(a) Evaluation
(i) Until formally rejected, prospective client is treated as current and the
same duties are owed regarding conflict, confidence, and competence.
1. If you reject, they become a former prospective client.
(ii) Do not have to accept a case in order to create a duty.
1. If you tell prospective client you are going to do something, you will
be liable for not doing what you said.
(iii)
Be clear and say in unambiguous terms that you are NOT taking
the case.
(b) Work
(i) Specify the scope of representation.
(ii) Client is your client until
1. You complete the services you promised.
2. You withdraw as specified under MR 1.16
3. You are fired by the client.
(c) Follow-up
(i) Expectation of follow-up services is viewed from clients perspective.
1. Such expectation maintains clients status as current client, even if
communication is not expressed to you.
a. E.g., reasonable to expect that you monitor other party adhering to
compliance with settlement terms.
(d) Pattern of work
(i) Arises when client has retained you to do legal work often enough to
establish a pattern of relationship (client can say thats my lawyer), even
if not doing work for client at that moment.
ii) Former
(1) Anyone who was ever your client in the past, including both individuals, and
entities, whether you served them at your current law firm or at some other former
legal job.
(2) The rules governing successive conflicts are far more lenient than the rules
governing concurrent conflicts, so former clients have less protection than current
clients.
iii) Never
(1) Everyone who is neither current nor former.
(a) Organizationsdont automatically represent officers, directors, employees
(b) Parents and childrenif you represent a child, you dont automatically
represent the parents
21

(c) Third person paying a feeif someone is paying a fee for your client, you
dont automatically represent that person
e) Direct Conflictsbetween two current clients
i) Can arise in five ways:
(1) Representing opposing sides in the same litigation
(a) Universally forbidden, regardless of client consent
(2) Representation opposing sides in a transaction
(a) Not prohibited by MR 1.7 but risky
(3) Opposing a current client in unrelated litigation
(a) Consentable but absolute right to refuse to consent to being opposed in any
litigation matter, and if current client consents, lawyer must refuse to accept
new case.
(b) Need informed consent.
(4) Opposing a client in an unrelated transaction
(a) Consentable
(i) E.g., if a lawyer in a asked to represent the seller of a business in
negotiations with a buyer who is currently represented by the lawyer, not
in same transaction, but in an unrelated matter, lawyer needs informed
consent.
(b) Comment 6 to MR 1.7:
(i) Simultaneous representation in unrelated matters of clients whose
interests are only economically adverse . . . does not ordinarily constitute a
conflict of interest and thus may not require consent of the respective
clients.
(5) Multiple Representation of allied parties
(a) It is always consentable.
(i) In civil, there is a limited scope under MR 1.2(c) so if clients interests
diverge during settlement, attorney can withdraw.
(b) To get consent, lawyer must explain:
(i) Implications of common representation in writing
1. Including the effects on loyalty, confidentiality, attorney-client,
privilege and advantages and risks involved.
a. Attorney client privilege does not attach to commonly represented
clients, and neither can claim if representation fails.
2. If one client decides that some material matter be kept from the other,
lawyer will have to withdraw because there is a duty of lyallty to each
client, and each client has the right to know about anything that might
affect clients interests.
3. If representation fails due to irreconcilable conflict between parties,
lawyer will be forced to withdraw and there may be additional costs,
embarrassment and recrimination.
(c) Advantages:
(i) Save legal fees
22

(ii) Reduces delayed because of scheduling


(iii)
If dispute does arise, common lawyer may help parties work out
differences before problem erupts into litigation or ruins a deal.
(d) Disadvantages:
(i) If conflict develops, common lawyer may ignore conflict or deliberately
conceal it so lawyer can continue multiple representation.
(ii) If serious, insoluble conflict arises, all parties may have to get new
lawyers, resulting in additional time and expense.
(iii)
If common clients get into dispute, they will not be able to claim
attorney-client privilege for communications with lawyer during common
representation.
1. Attorney must advise clients of this waiver.
(e) Criminal Defense Work
(i) ABA Model Rules 1.7:
1. The potential for conflict of interest in representing multiple
defendants in a criminal case is so grave that ordinarily a lawyer
should decline to represent more than one co-defendant.
(ii) It is a good idea to disclose everything to the judge to make sure he is
okay with multiple representation.
(iii)
Prosecution will be hesitant because it could result in ineffective
assistance of counsel.
f) Indirect Conflicts
i) Lawyer represents different clients in different matters and one matter may adversely
affect the other.
(1) Although clients are not directly attacking or opposing each other, one client
either:
(a) Wants something that would or could harm the other client; or
(b) Opposes something that would or could help the other client.
ii) Materially limiting conflict; not directly adverse.
g) Position Conflicts
i) It may be a conflict to represent different parties in completely different matters if
their legal positions are incompatible.
ii) A lawyer may take inconsistent legal positions in different tribunals at different times
on behalf of different clients.
(1) Merely, advocating a legal position for one client that might create a precedent
harmful to another client in an unrelated matter does not create a conflict.
iii) A conflict does exist if there is a significant risk that a lawyers action on behalf of
one client will create a precedent likely to seriously weaken the position taken on
behalf of the other client.
(1) Factors to be considered:
(a) Where are the cases pending?
(b) Substantive or procedural?
23

(c) Temporal relationship?


(d) Significant of the issues?
(e) Clients expectations?
(i) If significant risk of material limitation, informed consent is needed.
h) Conflicts Between Client and 3rd Party:
i) Directly Opposing 3rd Party
(1) A lawyer may be unwilling to oppose the person even though that person should
be a defendant in the litigation.
(2) A lawyer may be willing to oppose the third person, but may be unable or
unwilling to press the clients cause with maximum vigor against that person.
ii) Adversely Affecting a 3rd Party
(1) A lawyer may believe that adding a particular bank as a defendant in a loan fraud
case may anger or implicate his sister if she is an office of the bank.
(2) A personal injury lawyer defending a client in an accident case may believe that
the best defense is to blame one of the lawyers good friends for causing the
accident.
i) Conflicts with Lawyers Own Interests
i) Lawyers interests should not be permitted to have an adverse effect on
representation.
(1) E.g., if lawyer knows handling a matter would require traveling and he wants to
stay home for a weddingdo not accept.
ii) Business Transactions with Client:
(1) MR 1.7(b) and 1.8(a) prohibit lawyer from a lawyer from entering into business
deal with a client unless lawyer satisfies the following 3 conditions:
(a) Transaction and terms are fair and reasonable to client and are fully disclosed,
and transmitted in writing in a manner that can be reasonably understood by
client.
(b) Client is advised in writing of desirability of seeking advice of independent
counsel on transaction.
(c) Client gives informed consent, in writing signed by the client, to the essential
terms of the transaction and lawyers role in the transaction, including whether
lawyer is representing the client in the transaction.
(2) Highly risk for lawyers because failure to follow MR 1.8(a) can result in suit by
client against lawyer for fraud, misrepresentation, and breach of fiduciary duty.
(a) Disbarment and length suspensions are common penalties.
(3) In many jurisdictions, business transactions between lawyer and client are
MR 1.7Conflict
of Interest:fraudulent
Current Clients
presumptively
or improper.
(a) Except as(a)
provided
in
paragraph
(b),
a
lawyer
shallto
notshow
represent
client if the representation
involves
a concurrent
Burden of lawyer is on lawyer
thatatransaction
is fair and client
was
conflict of interest. A concurrent conflict of interest exists if:
fully informed
allbeterms
in adverse
writingtobefore
(1) the representation
of one clientofwill
directly
anotherentering
client; or the deal.
(2) there is a significant risk that the representation of one or more clients will be materially limited by the lawyers
responsibilities
to another
client,Current
a formerClients:
client orSpecific
a third person
MR
1.8Conflict
of Interest:
Rules or by a personal interest of the lawyer.
(b)
Notwithstanding
existence
of a concurrent
conflict
paragraph
(a), an
a lawyer
may represent
a client
(a) A
lawyer shall notthe
enter
into a business
transaction
withofa interest
client orunder
knowingly
acquire
ownership,
possessory,
if:
security or other pecuniary interest adverse to a client unless:
(1) the lawyer
reasonably
believes
that the lawyer acquires
will be able
provide
and diligent
representation
to each
transaction
and terms
on which
the to
interest
arecompetent
fair and reasonable
to the
client and are
fully
24 can be reasonably understood by the client;
affected
disclosedclient;
and transmitted in writing in a manner that
(2) the client
representation
notwriting
prohibited
law;
is advisedisin
of thebydesirability
of seeking and is given a reasonable opportunity to seek the advice
(3)independent
the representation
does not
the assertion
of
legal counsel
oninvolve
the transaction;
and of a claim by one client against another client represented by the
lawyer
the same
other proceeding
before
a tribunal;
and to the essential terms of the transaction and the
(3) theinclient
giveslitigation
informedorconsent,
in a writing
signed
by the client,
(4) each affected
client
gives informed
consent,
confirmed
in writing.
lawyers
role in the
transaction,
including
whether
the lawyer
is representing the client in the transaction.

VI) Criminal CasesLimits on Zealous Representation


a) Litigation TacticsTrial Publicity
i) Media coverage can influence jurors, witnesses and judges.
ii) Press has virtually unlimited right to publish what it believes is important about legal
proceedings.
iii) Courts have broad powers to regulate conduct of anyone who appears before the
court; including the press.
(1) Sheppard v. Maxwellcourt focused on trial judges failure to control the media
iv) Attorney Dilemma:
(1) Office of the courtprotecting the integrity of the proceedings
(2) Client representationprimary obligation is to clients
(a) MR 3.6(c)=right of reply
MR
3.6Trial
Publicity
v) Statements
made by prosecutors pose a greater risk than statements made by defender
(a) A lawyer who is participating or has participated in the investigation or litigation of a matter shall not make an
counsel.
extrajudicial
statement that the lawyer knows or reasonably should know will be disseminated by means of public
vi)
Tacticaland
decisions
left to discretion
attorneyeven
if client
doesnt proceeding
want you in the matter.
communication
will haveare
a substantial
likelihood of
of materially
prejudicing
an adjudicative
(b) Notwithstanding
paragraph (a), a lawyer may state:
to.

(1) the claim, offense or defense involves and, except when prohibited by law, the identity of the persons involved;
(2) information contained in a public record;
(3) that an investigation of a matter is in progress;
(4) the scheduling or result of any step in litigation;
(5) a request for assistance in obtaining evidence and information necessary thereto;
(6) a warning of danger concerning the behavior of a person involved, when there is reason to believe that there exists
the likelihood of substantial harm to an individual or to the public interest; and
(7) in a criminal case, in additional to subparagraphs (1) through (6);
(i) the identity, residence, occupation and family status of the accused;
(ii) if the accused has not been apprehended, information necessary to aid in apprehension of that person;
(iii) the fact, time and place of arrest; and
(iv) the identity of investigating and arresting officers or agencies and the length of the investigation.
(c) Notwithstanding paragraph (a), a lawyer may make a statement that a reasonable lawyer would believe is required to
protect a client from the substantial undue prejudicial
25 effect of recent publicity not initiated by the lawyer or the lawyers
client. A statement made pursuant to this paragraph shall be limited to such information as is necessary to mitigate the
recent adverse publicity.
(d) No lawyer associated in a firm or government agency with a lawyer subject to paragraph (a) shall make a statement
prohibited by paragraph (a).

b) Improper Contacts
i) Ex parte contactslawyers prohibited from speaking to jurors/witnesses/judges
without other attorney present (including emails).
ii) Conversations with jurors about facts of a case are improper.
(1) Saying hi to jurors in the hallways is improper.
(2) If juror says hi to youtell the judge to cover your ass.
MR 3.5Impartiality and Decorum of the Tribunal
A lawyer shall not:
(a) seek to influence a judge, juror, prospective juror or other official by means prohibited by law;
(b) communicate ex parte with such a person during the proceeding unless authorized to do so by law or court order;
(c) communicate with a juror or prospective juror after discharge of the jury if:
(1) the communication is prohibited by law or court order; or
(2) the juror has made known to the lawyer a desire not to communicate; or
(3) the communication involves misrepresentation, coercion, duress or harassment; or
(d) engage in conduct intended to disrupt a tribunal.

c) Duty to disclose Adverse Facts and Law


i) Lawyer does not have duty to help adversary from making mistakes.
(1) However, if opponent is presenting legal argument and facts and fails to cite
controlling authority in that jurisdiction for position that would be directly
adverse to your side, you must bring it before the court.
(a) The more unhappy you are with that case, the clearer your obligation to reveal
it.
(b) Controlling=same state or district/circuit
d) Fairness to Opposing Party and Counsel
MR 3.4Fairness to Opposing Party and Counsel
A lawyer shall not:
(a) unlawfully obstruct another partys access to evidence or unlawfully alter, destroy or conceal a document or other
material having potential evidentiary value. A lawyer shall not counsel ro assist another person to do such act;
(b) falsify evidence, counsel or assist a witness to testify falsely, or offer an inducement to a witness that is prohibited
by law;
(c) knowingly disobey an obligation under the rules of a tribunal, except for an open refusal based on an assertion
that no valid obligation exists;
(d) in pretrial procedure, make a frivolous discovery request or fail to make reasonably diligent effort to comply with
a legally proper discovery request by an opposing party;
(e) in trial, allude to any matter that the lawyer does not reasonably believe is relevant or what will not be supported
by admissible evidence, assert personal knowledge of facts in issue except when testifying as a witness, or state a
personal opinion as to the justness of a cause, the credibility of a witness, the culpability of a civil litigant or the guilt
or innocent of an accused; or
(f) request a person other than a client to refrain from voluntarily giving relevant information to another party unless:
(1) the person is a relative or an employee or other agent of a client; and
(2) the lawyer reasonably believes that the persons interests will not be adversely affected by refraining from
giving such information.

26

e) Respect for Rights of Third Parties


MR 4.4Respect for Rights of Third Persons
(a) In representing a client, a lawyer shall not use means that have no substantial purpose other than to embarrass, delay,
or burden a third person, or use methods of obtaining evidence that violate the legal rights of such a person.
(b) A lawyer who receives a document relating to the representation of the lawyers client and knows or reasonably
should know that the document was inadvertently sent shall promptly notify the sender.

f) Special Duties of Prosecutors


i) Obligation to do Justice
(1) Prosecutors are subject to state ethics rules.
(a) Federal prosecutors are subject to ethics rules of the state in which the district
court sits.
ii) Disclosure of Exculpatory Evidence
(1) Disclosing exculpatory evidence to grand jury is discretionary.
(a) Department of Justice policy says it must disclose or it violates the rules of
professional responsibility.
(b) Exculpatory tends to negate guilt or prove innocence
(c) Material evidence reasonable probability of affecting outcome
iii) Bradysuppression by prosecution of evidence favorable to an accused upon request
violates due process where the evidence is material to either guilt or punishment,
irrespective of good or bad faith of the prosecution
(1) Applies even if prosecution has not suppressed evidence and defense did not file
Brady motionmust turn over if you know
iv) Communication with person Represented by Counsel
(1) United States v. Talaoprosecutor talking with someone represented
(a) What factors are relevant in determining whether communications preindictment are authorized by law (post indictment communications are not
authorized):
(i) Has there been development of an adversarial relationship?
(ii) Did the informant or government initiate communication?
(iii)
Does the communication involved demonstrate evidence of
subordination of perjury or obstruction of justice?
(iv)Does defense counsel have a conflict of interest because informant offered
evidence of criminal activity adverse to interests of other clients
represented by counsel?
(v) Has the government given appropriate warnings of the persons right to
retain substitute counsel?
(vi)Has the government engaged in misconduct in connection with the
communication, such as misrepresentation, coercion, or attempts to obtain
attorney-client communications?

27

MR 3.8Special Responsibilities of a Prosecution


The prosecutor in a criminal case shall...
(a) refrain from prosecuting a charge that the prosecutors knows is not supported by probable cause;
(b) make reasonable efforts to assure that the accused has been advised of the right to, and the procedure for obtaining,
counsel and has been given reasonable opportunity to obtain counsel;
(c) not seek to obtain from an unrepresented accused a waiver of important pretrial rights, such as the right to a preliminary
hearing;
(d) make timely disclosure to the defense of all evidence or information known to the prosecutor that tends to negate the guilt
of the accused or mitigates the offense, and, in connection with sentencing, disclose the defense and to the tribunal all
unprivileged mitigating information known to the prosecutor, except when the prosecutor is relieved of this responsibility by
a protective order of the tribunal;
(e) not subpoena a lawyer in a grand jury or other criminal proceeding to present evidence about a past or present client
unless the prosecutor reasonable believes:
(1) the information sought is not protected from disclosure by any applicable privilege;
(2) the evidence sought is essential to the successful completion of an ongoing investigation or prosecution; and
(3) there is no other feasible alternative to obtain the information;
(f) except for statement that are necessary to inform the public of the nature and extent of the prosecutors action and that
serve a legitimate law enforcement purpose, refrain from making extrajudicial comments that have a substantial likelihood of
heightening public condemnation of the accused and exercise reasonable care to prevent investigators, law enforcement
personnel, employees or other persons assisting or associated with the prosecutor in a criminal case form making an
extrajudicial statement that the prosecutor would be prohibited from making under Rule 3.6 or this Rule.
(g) when a prosecutor knows of new, credible and material evidence creating a reasonable likelihood that a convicted
defendant did not commit an offense of which the defendant was convicted, the prosecutor shall:
(1) promptly disclose that evidence to an appropriate court or authority, and
(2) if the conviction was obtained in the prosecutors jurisdiction,
(A) promptly disclose that evidence to the defendant unless a court authorizes delay, and
(B) undertake further investigation, or make reasonable efforts to cause an investigation, to determine whether the
defendant was convicted of an offense that the defendant did not commit.
(h) When a prosecutor knows of clear and convincing evidence establishing that a defendant in the prosecutors jurisdiction
was convicted of an offense that the defendant did not commit, the prosecutor shall seek to remedy the conviction.

VII) Civil CasesClient-Lawyer Relationship: Fees (MR 1.5)


a) Two of the most frequent fees:
28

i) Contingent (Plaintiff)usually a percentage of what they obtain


(1) Lawyer collects only if matter is resolved in Ps favor.
(2) Not allowed in criminal and domestic relations cases.
(3) Not allowed in large settlement cases resulting in billion dollar verdicts.
(a) Also not allowed in cases where it is certain the client will win.
(4) Requirements:
(a) Must be in writing and signed by the client
(i) Stating how fee is calculated
(ii) Explaining other expenses (depositions, travel etc.)
(iii)
When expenses are calculated (before or after the fee is taken)
(iv)After contingency fee is charged lawyer must give written completion to
the client
(b) Time and labor required
(c) Fee customarily charged in that area
(d) Amount involved and results obtained
(5) The total fee charges must be reasonable.
(6) Some states have statutory requirements for contingency fees.
(a) Standard is 33% before expenses.
(b) Expert witnesses cannot be paid on contingent basis.
ii) Hourly (Defendant)Insurance defense
(1) Cannot double bill airplane example
b) Fee splitting:
i) MR 1.5 says lawyers cannot be compensated for assuming an entirely passive role.
ii) Total fee must be reasonable.
iii) Client must agree to fee splitting in writing
iv) Joint responsibility or proportion to services renderedfully liable in malpractice
action
c) Comingling Funds
i) Lawyers cannot comingle with personal funds; must maintain separate trust account
and keep it up to date.
(1) Settlement goes into trust account then lawyer writes check to personal account;
any other way is dangerous.
(a) Trust account Pay proceeds client Pay lawyer into business account
(b) Cannot write check from trust account to cover business expenses (e.g., yearly
bar fee).
ii) Comingling of funds is the single most popular reason for disbarment.
(1) Per Se offense intention doesnt matter; some states it is automatic disbarment
d) Telex
i) T hires Lasky a prominent antitrust attorney to petition Supreme Court
(1) Hired on contingency fee basis--$25K retainer, if writ is denied then no settlement
beyond
29

(a) If petition is filed and T settles, L gets 5% of net recovery but not less than
$1 million.
(b) There was a settlement but no one paid anything, all T did was file to Supreme
Court.
(2) Was the contact between client and lawyer unconscionable?
(a) No!lawyer got $1 million
e) Matter of Laurence Fordham
i) Fordham was very experienced and hired to handle a case he had no dealt with
before.
(1) He told client that he had never done it before and he works on hourly fee basis.
(2) He was able to win and get an acquittal for his client.
(3) Lawyer charged $40,022.25 for 227 hours of court.
(4) Court finds the fee to be excessive
(a) Under MR 1.5(a)
(i) Time requiredlawyer spent more than what a prudent experience lawyer
would have had
(ii) Customary feeother lawyers charged 1/3 as much
(b) Client cannot be held to have understood such an unreasonable feenever
given estimate like in Telex.

VIII) Clients Role in Adversarial System


a) Questions:
i) Who decides what measures a lawyer takes?
ii) How do we allocate power between lawyer and client?
iii) Who controls the way a case is handled?
b) Allocation Power Between Attorney and Client
i) Model Rules explain (MR 1.2):
(1) Objective=controlled by client
(2) Means=controlled by lawyer (after consulting with the client, MR 1.4)
(3) Lawyer may limit scope of representation if limitation is reasonable and there is
informed consent.
30

(a) It is unreasonable if the time allotted was not sufficient to yield independent
advice of other counsel.
ii) Decisions reserved to client
(1) In civil matters, a lawyer must abide by a clients decision whether to accept (or
reject) an offer of settlement.
(2) In criminal matters, a lawyer must abide by a clients decision as to:
(a) Entering a plea, waiving a jury trial or testifying.
c) Jones v. Barnes
i) Barnes convicted of several felonies, appealed conviction and was assigned attorney.
(1) He told attorney to raise some non-frivolous arguments but attorney only raised
three of them.
ii) Barnes raised all of them in a pro se brief.
(1) Appellate court rejected all argument and confirmed conviction.
(a) Barnes claimed ineffective assistance of counsel.
iii) Court held that: Constitution grants accused the authority to make fundamental
decisions on pleading guilty, waiving a jury trial, testifying, and whether or not to
appeal.
(1) But, the attorney has the power to decide what issues to raise on appeal.
iv) Dissent argued that client has right to be unwise and dictate which non-frivolous
arguments to raise.

MR 1.4Communication
(a) A lawyer shall:
(1) promptly inform the client of any decision or circumstance with respect to which the clients informed consent, as
defined in Rule 1.0(e), is required by these Rules;
(2) reasonably consult with the client about the means by which the clients objectives are to be accomplished;
(3) keep the client reasonably informed about the status of the matter;
(4) promptly comply with reasonable requests for information; and
(5) consult with the client about any relevant limitation on the lawyers conduct when the lawyer knows that the client
expects assistance not permitted by the Rules of Professional Conduct or other law.
(b) A lawyer shall not explain a matter to the extent reasonably necessary to permit the client to make informed
decisions regarding the representation.
MR 1.2Scope of Representation and Allocation of Authority Between Client and Lawyer
(a) Subject to paragraphs (c) and (d), a lawyer shall abide by a clients decisions concerning the objectives of
representation and, as required Rule 1.4, shall consult with the client as to the means by which they are pursued. A
lawyer may take such action on behalf of the client as is impliedly authorized to carry out the representation. A lawyer
shall abide by a clients decision whether to settle a matter. In a criminal case, the lawyer shall abide by the clients
decision, after consultation with the lawyer, as to a plea to be entered, whether to waive jury trial and whether the client
will testify.
(b) A lawyers representation of a client, including representation by appointment, does not constitute an endorsement
of the clients political, economic, social or moral views or activities.
(c) A lawyer may limit the scope of the representation if the limitation is reasonable under the circumstances and the
31
client gives informed consent.
(d) A lawyer shall not counsel a client to engage, or assist a client, in conduct that the lawyer knows is criminal or
fraudulent, but a lawyer may discuss the legal consequences of any proposed course of conduct with a client and may
counsel or assist a client to make a good faith effort to determine the validity, scope, meaning or application of the law.

d) Terminating Attorney-Client Relationship


i) Mandatory Withdrawal (MR 1.16(a))
(1) Ethical rule or law would be violated.
(2) Physical or mental condition materially impairs lawyers ability to represent
client.
(3) Client discharges attorney.
ii) Permissive Withdrawal (MR 1.16(b))
(1) Can be accomplished without materially adverse effect.
(2) Client pursues illegal course or used services to perpetrate crime or fraud.
(3) Client pursues an objective the lawyer finds repugnant or with which the lawyer
has a fundamental disagreement.
(4) Client fails to pay fees.
(5) Attorney would suffer unreasonable financial burden.
(6) Other good cause.
iii) Attorneys Duty Upon Termination
(1) If the matter is pending before a tribunal, the attorney must have permission of the
tribunal.
(2) Attorney must take steps to ensure no prejudice to client.
iv) Non-Engagement Letter
(1) Togstad v. Vesley/Miller
(a) Attorney-client relationship taken from perspective of client.
(b) Lawyer has burden of clarifying whether representation has been taken.
(c) Lawyer should try to record any legal advice given to a client.
MR 1.16Declining or Terminating Representation
(d) Non-engagement letter should be clear:
(a) Except as stated in paragraph (c), a lawyer shall not represent a client or, where representation has commenced,
(i) the
Unambiguously
youif:are not taking the representation.
shall withdraw from
representation ofsay
a client
(ii)
Notify
non-client
(prospective
that they
should
seeklaw;
advice of other
(1) the representation will result in violation of the rules client)
of professional
conduct
or other
(2) the lawyers physical
or
mental
condition
materially
impairs
the
lawyers
ability
to
represent
the client; or
counsel.
(3) the lawyer is discharged.
(iii)
Caution client if there is a statute of limitations.
(b) Except as stated in paragraph (c), a lawyer may withdraw from representing a client if:
(iv)Give
vague
reason
why
you arent
it.the interests if the client;
(1) withdrawal can be accomplished
without
material
adversetaking
effect on
(v) Doin not
say too
much
(e.g., likelihood
of recovery
not sufficient.)
(2) the client persists
a course
of action
involving
the lawyers services
that theislawyer
reasonably believes is
criminal or fraudulent;
(3) the client has used the lawyers services to perpetrate a crime or fraud;
(4) the client insists upon taking action that the lawyer considers repugnant or with which the lawyer has a
fundamental disagreement;
(5) the client fails substantially to fulfill an obligation to the lawyer regarding the lawyers services and has been
given reasonable warning that the lawyer will withdraw unless the obligation is fulfilled;
(6) the representation will result in an unreasonable financial burden on the lawyer or has been rendered
unreasonably difficult by the client; or
(7) other good cause for withdrawal exists.
(c) A lawyer must comply with applicable law requiring notice to or permission of a tribunal when terminating a
representation. When ordered to do so by a tribunal, a lawyer shall continue representation notwithstanding good
cause for terminating the representation.
(d) Upon termination of representation, a lawyer shall
32 take steps to the extent reasonably practicable to protect a
clients interests, such as giving reasonable notice to the client, allowing time for employment of other counsel,
surrendering papers and property to which the client is entitled and refunding any advance payment of fee or expense
that has not been earned or incurred. The lawyer may retain papers relating to the client to the extent permitted by
other law.

IX) Civil Cases: Confidentiality


Protection

Arises From

A lawyer cannot..

Duty of Confidentiality

Rules of Legal Ethics

voluntarily tell other people anything about a client


without the clients consent, or some other exception

Attorney-Client Privilege

Rules of Evidence

be compelled to reveal communications with a client

Work Product Doctrine

Rules of Procedure

be compelled to reveal written material prepared in


anticipation of litigation.

a) Duty of Confidentiality
i) Secretaries are agents of lawyers and thus communications with them have protection
as confidentiality information.
ii) Never disclose confidential information without express authority of client.
iii) During negotiations you can disclose confidential information if it will advance your
clients position.
b) Attorney-Client Privilege
i) Five Cs: Client Communicates Confidentially with Counsel to obtain Counsel
(1) If covered by ACP then covered by duty of confidentiality.
c) Work Product Doctrine
d) Internal Investigations
33

i) Attorney-Client Privilege for Organizations


(1) Control group test (City of Philadelphia v. Westinghouse)
(a) Control group elite group of corporate officers who control corporation and
make decisions
(b) Problemslimits privilege to those who have control power over entity
(i) Too narrow because it fails to extend privilege to some people who are
speaking for the corporation (under inclusive).
(2) Subject matter test (Upjohn v. United States)
(a) Upjohn accused of bribing foreign officials.
(i) Upjohns general counsel conducted internal investigation.
(ii) Government sought to subpoena attorneys notes.
(b) Supreme Court found that conversations between a corporate attorney and any
corporate employeeeven if the conversations are initiated by the attorney
are privileged as long as the conversations are for the purpose of gathering
information necessary to give or implement legal advice to the corporation.
(c) Test focuses on the nature of the communication, not the status of the
communicator.
(d) Criticized for being too broad because it extends the privilege to people who
are merely witnesses and are not speaking for the corporation.
(3) Entity Client
(a) Samaritan Foundation v. Goodfarb
(i) If employee initiates conversation that is always privileged.
(ii) If someone else initiates then privilege sometimes applies.
1. Court will distinguish between employees who are mere witnesses and
those who are clients.
(iii)
True clients are those employees whose actions could result in
corporate liability.

34

X)Civil CasesConflicts of Interest


a) Concurrent and Successive Conflicts
i) Concurrent Conflict
(1) Under MR 1.7(a) a lawyer may not oppose a current client without that clients
consent even in a matter wholly unrelated to the lawyers other work for that
client.
(a) Under MR 1.7, you must always obtain consent to oppose a current client.
(b) Substantial relationshipinvolve the same transaction or legal dispute; or
substantial risk that confidential information would materially advance clients
position.
ii) Successive Conflict
(1) Under MR 1.9 you are not barred from opposing a former client unless the matter
is substantially related to your earlier work (or your firms earlier work) for the
former client.
(a) You do not need the former clients consent unless the matter is substantially
related.
(2) Courts will presume you received confidential information when you represented
a former client.
(a) This is a rebuttable presumption.
(i) E.g.m if you were an associate at a large firm who never worked on the
clients matters, never met with the former client, and has access only to
files you actually worked on (as opposed to all the firms files), then you
can probably rebut the presumption.
b) Grounds for Personal Disqualification
i) May be personally disqualified if:
(1) You personally represented the former client at your current law firm or in any
previous legal job, (MR1.9(a)); or
(2) You did not personally represent the former client, but, while at a previous legal
job, you acquired protected information about the former client in the same or
substantially related matter (MR 1.9(b)).
ii) A former client is always free to consent to a conflict whether it arises under
MR 1.9(a) or MR 1.9(b).
(1) Burden is NOT on the former client to object.
35

c) Conflict Checking
i) Under MR 1.9(a)
(1) New firms will likely make a condition offer:
(a) Show you currently active matters.
(b) Your former and current clients must be included in conflict check (including
your work as a paralegal/legal assistant).
(c) Client check encompasses all types of matters (litigation etc.)
(d) New firms will ask if your matters are adverse to their clients.
(e) New firm will ask old firm to seek consent from clients (cannot contact them
directly).
(i) If client of old firm refuses consent:
1. Firm can hire you and withdraw from matter
2. Make non-frivolous argument they may hire and run the risk.
3. Firm cannot hire you at all.
4. Firm can screen/firewall you from certain matters.
ii) Under MR 1.9(b)
(1) Even thought you never did any legal work for a particular client at your old firm,
did you nevertheless obtain confidential information about the client?
(a) This conflict check is a delicate/difficult task.
d) Firewall metaphorical wall of separation between lawyer with conflict and rest of the
firm.
i) Disqualified lawyer must be personally and completely isolated.
ii) Other lawyers must not discuss case with lawyer who is disqualified and eb careful
where they discuss cases.
iii) Firm should circulate a memo.
iv) Firm should make file inaccessible to disqualified attorney.
v) Firewalls are only used with former clients NOT currant clients.
vi) Positivesgive attorney freedom/mobility/flexibility.
vii) Negativesbreaches can occur inadvertently or maliciously.
MR 1.9Duties to Former Clients
(a) A lawyer who has formerly represented a client in a matter shall not thereafter represent another person in the
same or a substantially related matter in which that persons interests are materially adverse to the interests of the
former client unless the former client gives informed consent in writing.
(b) A lawyer shall not knowingly represent a person in the same or a substantially related matter in which a firm with
which the lawyer formerly was associated has previously represented a client
(1) whose interests are materially adverse to that person; and
(2) about whom the lawyer had acquired information protected by Rules 1.6 and 1.9(c) that is material to the matter;
Unless the former given gives informed consent, confirmed in writing.
(c) A lawyer who has formerly represented a client in a matter or whose present or former firm has formerly
represented a client in a matter shall not thereafter:
(1) use information relating to the representation to the disadvantage of the former client except as these Rules
would permit or require with respect to a client, or when the information has become generally known; or
(2) reveal information relating to the representation except as these Rules would permit tor require with respect to a
client.

e) Imputation Principle
36

f)

MR 1.10Imputation of Conflict of Interest: General Rule


(a) while lawyers are associated in a firm, none of them shall knowingly represent a client when any one of them
practicing alone would be prohibited from doing so by Rules 1.7 or 1.9, unless
(1) the prohibition is based upon a personal interest of the disqualified lawyer and does not present a significant
risk of materially limiting the representation of the client by the remaining lawyers in the firm; or
(2) the prohibition is based upon Rule 1.9(a) or (b), and arises out of the disqualified lawyers association with a
prior firm, and
(i) the disqualified lawyer is timely screened from any participation in the matter and is apportioned no part
of the free therefrom;
(ii) written notice is promptly given to any affected former clients to enable the former client to ascertain
compliance with the provisions of this Rule, which shall include a description of the screening procedures
employed; a statement of the firms and of the screened lawyers compliance with these Rules; a statement
that review may be available before a tribunal; and an agreement by the firm to respond promptly to any
written inquiries or objections by the former client about the screening procedures; and
(iii) certifications of compliance with these Rules and with the screening procedures are provided to the
former client by the screened lawyer and by a partner of the firm, at reasonable intervals upon the former
clients written request and upon termination of the screening procedures.
(b) When a lawyer has terminated an association with a firm, the firm is not prohibited from thereafter representing
a person with interests materially adverse to those of a client represented by the formerly associated lawyer and not
currently represented by the firm, unless
(1) the matter is the same or substantially related to that in which the formerly associated lawyer represented the
client; and
(2) any lawyer remaining in the firm has information protected by Rules 1.6 and 1.9(c) that is material to the
matter.
(c) A disqualification prescribed by this rule may be waived by the affected client under the conditions stated in
Rule 1.7.
Advocate
Witness Rule
(d) The disqualification
of lawyers associated in a firm with former or current government lawyers is governed by
Rule
1.11.
i) Applies any time attorney will be called as a party.

ii) Rationale:
(1) A jury may accord lawyers testimony too much weight because of their special
knowledge of the case.
(2) Professional courtesy may handicap the opposing lawyer on cross examination.
(3) The bar is ill served when attorneys veracity becomes an issue in the case.
(4) Jury might not distinguish lawyers role as advocate and witnesstestimony may
be given weight as a closing argument.
MR 3.7Lawyer as Witness
(a) A lawyer shall not act as advocate at a trial in which the lawyer is likely to be a necessary witness unless:
(1) the testimony relates to an uncontested issue;
(2) the testimony relates to the nature and value of legal services rendered in the case; or
(3) disqualification of the lawyer would work substantial hardship on the client.
(b) A lawyer may act as advocate in a trial in which another lawyer in the lawyers firm is likely to be called as a
witness unless precluded from doing so by Rule 1.7 or Rule 1.9

g) Multiple Parties
i) As long as co-plaintiffs arent suing each other then representation is allowed.
ii) Conflict can arise if specific discrepancy between co-parties testimony.
iii) Attorney should interview each party separately first.
(1) MR doesnt state who to interview first but it is logical to interview the one with a
great chance at more money first.
37

iv) Attorney must describe advantages and disadvantages to co-parties.


MR 1.18Duties to Prospective Client
(a) A person who discusses with a lawyer the possibility of forming a client-lawyer relationship with respect to a
matter is a prospective client.
(b) Even when no client-lawyer relationship ensues, a lawyer who has had discussions with a prospective client shall
not use or reveal information learned in the consultation, except as Rule 1.9 would permit with respect to information
of a former client.
(c) A lawyer subject to paragraph (b) shall not represent a client with interests materially adverse to those of a
prospective client in the same or a substantially related matter if the lawyer received information from the prospective
client that could be significantly harmful to that person in the matter, except as provided in paragraph (d). If a lawyer
is disqualified from representation under this paragraph, no lawyer in a firm with which that lawyer is associated may
knowingly undertake or continue representation in such a matter, except as provided in paragraph (d).
(d) When the lawyer has received disqualifying information as defined in paragraph (c), the representation is
permissible if:
(1) both the affected client and the prospective client have given informed consent, confirmed in writing, or:
(2) the lawyer who received the information took reasonable measures to avoid exposure to more disqualifying
information than was reasonable necessary to determine whether to represent the prospective client; and
(i) the disqualified lawyer is timely screened from any participation in the matter and is apportioned no part
of the fee therefrom; and
(ii) written notice is promptly given to the prospective client.

h) Insured and Insurer


i) Duties:
(1) Insurer has an obligation to pay lawyer for certain types of lawsuits.
(a) Insurer has obligation to pay settlement/judgment within policy limits.
(2) Insured has obligation to pay premium policy
(a) Duty to notify insurer of an accident/claim that could lead to lawsuit.
(b) Duty to cooperate with counsel.
(3) Defense counsel has attorney-client relationship with two parties.
(4) Dual client doctrine counsel represents both company and person as long as the
interests coincide
ii) Who is the obligation to?
(1) Every jurisdiction says that lawyer has a primary obligation to the insured.
(2) Lawyer has to represent insure but keep insurance company happy because that is
who pays them.
(3) Insurance companies do not provide counsel for cross or counter-claims.
(4) In joint representation there is no attorney-client privilege.
iii) Reservation of rights letter
(1) Insurance company sends letter to insured, whereby insurer says, we will provide
defense for you, BUT if you lose and there are damages, we will have another
case to determine if you are covered.
(2) What if counsel learns about possible lack of coverage?
(a) Withdraw if it is irreconcilable.
(b) Inform both insurer and insured that this information exists, relying on
multiple-representation situation and claim that ACP doesnt exist.
(c) Keep mouth shut and not disclose to insurer because lawyer is representing
the insured as his client, not the company.

MR 1.8Conflict of Interest: Current Clients: Specific Rules


(f) A lawyer shall not accept compensation for representing a client from one other than the client unless:
38
(1) the client gives informed consent;
(2) there is no interference with the lawyers independence of professional judgment or with the clientlawyer relationship; and
(3) information relating to representation of a client is protected as required by Rule 1.6.

XI) Civil CasesLimitations on Zealous Representation


a) Frivolous Claims
i) Duty not to bring frivolous claims.
ii) Why should lawyer be subject to sanctions if it is clients choice?
(1) Does not justify because legal proceedings are different from purely private
conduct because societal resources are used in legal proceedings.
iii) FRCP 11no filing of a motion without good faith basis that is well grounded
(1) Allows for monetary recovery
(2) Obligations:
(a) Cause of action not filed with improper purpose (e.g., to harass).
(i) Look at purpose/effect of litigation.
(ii) Under MR 2.1 lawyer can say it is immoral to file an improper claim.
(b) Legal contentions are warranted by existing law or nonfrivolous argument for
extending, modifying or reversing existing law or for establishing new law.
(i) Investigate statute of limitations.
(ii) If you are arguing new cause of action, make research memo so you can
show it when you are met with frivolous claim.
39

(c) Factual allegations have evidentiary support or will after reasonable


opportunity for investigation/recovery.
iv) MR 3.1
(1) Does not provide for monetary recovery
MR 3.1Mritorious Claims and Contentions
A lawyer shall not bring or defend a proceeding, or assert or controvert an issue therein, unless there is a
basis in law and fact for doing so that is not frivolous, which includes a good faith argument for an
extension, modification or reversal of existing law. A lawyer for the defendant in a criminal proceeding, or
the respondent in a proceeding that could result in incarceration, may nevertheless so defend the proceeding
as to require that every element of the case be established.

b) Anti-Contact Rule (MR 4.2)


i) Lawyer is prohibited from talking to person about a matter if they know that person is
represented by counsel.
(1) Lawyer cannot turn his head to the obvious.
(2) Protects attorney-client privilege.
(a) Not waiveable by client.
(i) Only lawyer can consent to talk, not client.
(3) If lawyer instructs paralegal (agent of lawyer) to contact other party it is improper.
ii) Niesig v. Team I
(1) Construction worker injured.
(2) Court of Appeals held anti-contact rule protects (applies only to current
employees:
(a) Employees whose acts can be imputed to corporation for liability;
(b) Agents who act at direction of lawyers; and
(c) Control group.
(3) Steps to take:
(a) Meet and identify yourself, say who you represent.
(b) Ask other person if someone represents them, if so stop.
(c) Do not ask questions relating to communication between witness and counsel
for company.
(d) Do not conduct yourself in any way that is harassing/intimidating.
(e) Give employee summary of what information was given and have them sign
it.
MR 4.2Communications with Person Represented by Counsel
In representing a client, a lawyer shall not communicate about the subject of the representation with a
person the lawyer knows to be represented by another lawyer in the matter, unless the lawyer has the
consent of the other lawyer or is authorized to do so by law or a court order.

c) Inadvertent Disclosure:
i) Modern approach is to look at facts and circumstances to determine whether attorneyclient privilege applies (look at precautions taken).
ii) Traditional approach was that the privilege was lost because the purpose was gone.
d) Discovery Abuse
i) Types:
40

(1) Broad request


(a) E.g., interrogatories that asks a lot of questions within one question.
(2) Withholding information that other party should receive
(a) Objection on grounds of work-product or attorney-client privilege are not
proper if interrogatory calls for things that happened before lawsuit.
(b) Must indicate that information is protected, and create a log of all information
you are withholding with justificationno blanket objection.
(3) Abusive behavior (depositions)
ii) To avoid problems with depositions:
(1) Prepare your client
(2) Hall v. Clitoncourt adopted guidelines:
(a) At beginning of deposition, tell witness to ask deposing counsel for
clarifications, not their own counsel.
(b) Objections, except those which would be waived, shall be preserved.
(c) Counsel shall not direct or request witness not to answer unless objection has
been raised.
(d) Counsel shall not make objections or statements that suggest answer to
witness.
(e) Counsel and witness/clients shall not engage in private conferences on breaks
except for deciding whether to assert a privilege.
(f) Any conferences that do happen are subject to inquiry by deposing counsel.
(g) Any conferences must be noted on record by counsel who participated,
purpose and outcome should be included.
(h) Deposing counsel should provide to witnesss counsel a copy of all documents
shown to witness during deposition.
iii) What if partner asks you to certify a response that you believe is improper?
(1) MR 5.2(b)not a reasonable resolution.
(a) As officer of the court you must comply with ethical rules, despite what
supervising attorney tells you.
(2) Do not certify and report for misconduct under MR 8.3
e) Tape Recordings
i) Lawful recording without consent does not violate MR.
(1) BUT under MR 8.4(c), lawyer cannot engage in fraudulent/deceitful conduct.
(a) Secret recordings are not inherently misleading, but representation about
whether a conversation was being recorded is improper.
ii) NY Rule
(1) One party consent only one party to conversation has to consent to recording
for it to be ethically permissible
(2) Other states (CA/FL) say that two party consent is needed.
MR 4.4Respect for Rights of Third Persons
(a) In representing a client, a lawyer shall not use means that have no substantial purpose other than to
embarrass, delay, or burden a third person, or use methods of obtaining evidence that violate the legal rights of
such person.
(b) A lawyer who receives a document relating to the representation of the lawyers client and knows or
reasonably should know that the document was inadvertently sent shall promptly notify the sender.

41

XII) Civil CasesAlternative Dispute Resolution


a) Duty of Honesty
i) No affirmative duty to inform an opposing part of relevant facts.
ii) Misrepresentation can occur if lawyer affirms a statement of another person knowing
it is false.
iii) Not a statement of fact
(1) Generally acceptable negotiation conversationestimate of price or value of the
subject of a transaction and partys intentions regarding an acceptable settlement.
(a) E.g., counsel says P wants his job back when he really doesnt
misrepresenting what you are willing to settle for.
(b) E.g., CANNOT say discharge caused P to have emotional distress s(improper
misrepresentation of a claim).
b) Spaulding v. Zimmerman
i) Court said no abuse of discretion under rules of procedure.
ii) No case is important enough to violate the adversary system.
iii) Two lessons:
(a) Lawyer had duty to counsel their client. Must engage in dialogue about how
to act decently and morally.
(b) Sometimes things can get better. This case suggests that things have
improved.
iv) Holding:
(1) A settlement can be vacated under FRCP 60(b) when a party withholds medical
information unknown to their adversary that materially affects the negotiation.
(a) No duty to disclose BUT a settlement can be vacated if it relies on a unilateral
mistake that materially affected the negotiations.
c) Fairness of the settlement
i) Rules of ethics do not impose obligation on lawyers to refuse to participate in
agreements that are unfair because:
(1) No objective criteria of fairness;
(2) Rules impose obligation to avoid fraud and mistake which sort of covers this; and
42

(3) Duty of fairness is inconsistent with adversarial system.


MR 4.1Truthfulness in Statements to Others
In the course of representing a client a lawyer shall not knowingly:
(a) make a false statement of material fact or law to a third person; or
(b) fail to disclose a material fact to a third person when disclosure is necessary to avoid assisting a criminal or
fraudulent act by a client, unless disclosure is prohibited by Rule 1.6.

XIII) Delivery of Legal Services


a) Advertising and Solicitation
i) Definitions:
(1) Advertising non-targeted mass media (radio/TV/billboard)
(2) Solicitation targeted and personal recruiting (ambulance chasers)
ii) How do lawyers get clients?
(1) Large firms have three types:
(a) Findersgo out find clients and bring them in
(b) Minderssenior partners and associates that supervise
(c) Grindersfirst years that work hard
(2) Three ways to attract clients:
(a) Recommendationword of mouth
(b) Court appointments
(c) Advertisements and solicitation
iii) Bates v. State of Arizona
(1) Two lawyers rebelled against state law blanket prohibition against advertising.
(2) Advertising is constitutionally protected and could not be prohibited by state.
(a) BUT lawyer has no 1st Amendment right to use advertisements that are false or
misleading.
(b) Lawyer has no right to engage in in-person solicitation.
(3) Three principles to determine constitutionality of restrictions on legal advertising
(Central Hudson):
(a) A state may constitutionally prohibit lawyers from engaging in false or
inherently misleading advertising or when experience has proven that in fact
such advertising is subject to abuse.
(b) A state may not prohibit advertising that is potentially misleading, but may
regulate such advertising by a method no broader than reasonably necessary
to prevent the deception, such as by explanation or disclaimer.
(c) States retain the right to regulate truthful advertising in some limited
circumstances if the stat establishes a substantial interest and the interference
with speech must be in proportion to the interest served.
MR 7.1Communications Concerning a Lawyers Services
A lawyer shall not make a false or misleading communication about the lawyer or the lawyers services. A
communication is false or misleading if it contains a material misrepresentation of fact or law, or omits a fact
necessary to make the statement considered as a whole not materially misleading.

43

MR 7.2Advertising
(a) Subject to the requirements of Rules 7.1 and 7.3, a lawyer may advertise services through written recorded or electronic
communication, including public media.
(b) A lawyer shall not give anything of value to a person for recommending the lawyers services except that a lawyer may
(1) pay the reasonable costs of advertisements or communications permitted by this Rule;
(2) pay the usual charges of a legal services plan or a not-for-profit or qualified lawyer referral service. A qualified lawyer referral
service is a lawyer referral service that has been approved by an appropriate regulatory authority;
(3) pay for a law practice in accordance with Rule 1.17; and
(4) refer clients to another lawyer or a nonlawyer professional pursuant to an agreement not otherwise prohibited under these
Rules that provides for the other person to refer clients or customers to the lawyer, if
(i) the reciprocal referral agreement is not exclusive, and
(ii) the client is informed of the existence and nature of the agreement.
(c) Any communication made pursuant to this rule shall include the name and office address of at least one lawyer or law firm
responsible for its content.
MR 7.3Direct Contact with prospective Clients
(a) A lawyer shall not by in-person, live telephone or real-time electronic contact solicit professional employment from a
prospective client when significant motive for the lawyers doing so is the lawyers pecuniary gain, unless the person contacted:
(1) is a lawyer; or
(2) has a family, close personal, or prior professional relationship with the lawyer.
(b) A lawyer shall not solicit professional employment from a prospective client by written, recorded or electronic communication
or by in-person, telephone r real-time electronic contact even when not otherwise prohibited by paragraph (a), if:
(1) the prospective client has made known to the lawyer a desire not to be solicited by the lawyer; or
(2) the solicitation involves coercion, duress or harassment.
(c) Every written, recorded or electronic communication from a lawyer soliciting professional employment from a prospective
client known to be in need of legal services in a particular matter shall include the words Advertising Material on the outside
envelope, if any, and at the beginning and ending of any recorded or electronic communication, unless the recipient of the
communication is a person specified in paragraphs (a)(1) or (a)(2).
(d) Notwithstanding the prohibitions in paragraph (a), a lawyer may participate with a prepaid or group legal service plan operated
by an organization not owned or directed by the lawyer that uses in-person or telephone contact to solicit memberships or
subscriptions for the plan from persons who are not known to need legal services in a particular matter covered by the plan.
MR 7.4Communication of Fields of Practice and Specialization
(a) A lawyer may communicate the fact that the lawyer does or does not practice in particular fields of law.
(d) A lawyer shall not state or imply that a lawyer is certified as a specialist in a particular field of law, unless:
(1) the lawyer has been certified as a specialist by an organization that has been approved by an appropriate state authority or that
has been accredited by the American Bar Association; and
(2) the name of the certifying organization is clearly identified in the communication.
MR 7.5Firm Names and Letterheads
(a) A lawyer shall not use a firm name, letterhead or other professional designation that violates Rule 7.1. A trade name may be
used by a lawyer in private practice if it does not imply a connection with a government agency or with a public or charitable legal
services organization and is not otherwise in violation of Rule 7.1.
(b) A law firm with offices in more than one jurisdiction may use the same name or other professional designation in each
jurisdiction, but identification of the lawyers in an office of the firm shall indicate the jurisdictional limitations on those not
licensed to practice in the jurisdiction where the office is located.
(c) The name of a lawyer holding a public office shall not be used in the name of a law firm, or in communications on its behalf,
during any substantial period in which the lawyer is not actively and regularly practicing with the firm.
(d) Lawyers may state or imply that they practice in a partnership or other organization only when that is the fact.

b) Unauthorized Practice of Law/Multijurisdictional Practice


i) Practice by non-lawyers:
(1) A crime in most jurisdictions
44

(2) If lawyers assist non-lawyers, this is a violation of the MR.


ii) Practice by lawyers in a jurisdiction they are not admitted to:
(1) Used to be barred from this but now we have pro hac vice, in-house counsel and
associations with local counsel.
(2) Practicing law anything that can be characterized as the application of legal
training, judgment, and skill in advising or otherwise assisting another person to
analyze or solve the particular legal problem.
(3) Berbrower v. Superior Court
(a) California refused to enforce fees from NY law firm with no office in CA.
Contract dispute was governed by CA law in arbitration. Client ended up
filing for malpractice and unauthorized practice of law claim. If this had been
litigation could have had pro hac vice but cannot do that in arbitration.
(b) Lawyer practiced law here; you do not have to be in California to practice law.
There was sufficient contacts/activity.
MR 5.5Unauthorized Practice of Law; Multijurisdictional Practice of Law
(a) A lawyer shall not practice law in a jurisdiction in violation of the regulation of the legal profession in that jurisdiction,
or assist another in doing so.
(b) A lawyer who is not admitted to practice in this jurisdiction shall not:
(1) except as authorized by these Rules or other law, establish an office or other systematic and continuous presence in
this jurisdiction for the practice of law; or
(2) hold out to the public or otherwise represent that the lawyer is admitted to practice law in this jurisdiction.
(c) A lawyer admitted in another United States jurisdiction, and not disbarred or suspended from practice in any
jurisdiction, may provide legal services on a temporary basis in this jurisdiction that:
(1) are undertaken in association with a lawyer who is admitted to practice in this jurisdiction and who actively
participate in the matter;
(2) are in or reasonably related to a pending or potential proceeding before a tribunal in this case or another jurisdiction,
if the lawyer, or a person the lawyer is assisting, is authorized by law or order to appear in such proceeding or
reasonably expects to be so authorized;
(3) are in or reasonably related to a pending or potential arbitration, mediation, or other alternative dispute resolution
proceeding in this or another jurisdiction, if the services arise out of or are reasonably related to the lawyers practice in
a jurisdiction in which the lawyer is admitted to practice and are not services for which the forum requires pro hac vice
admission; or
(4) are not within paragraphs (c)(2) or (c)(3) and arise out of or are reasonably related to the lawyers practice in a
jurisdiction in which the lawyer is admitted to practice.
(d) A lawyer admitted in another United States jurisdiction, and not disbarred or suspended form practice in any
jurisdiction, may provide legal services in this jurisdiction that:
(1) are provided to the lawyers employer or its organizational affiliates and are not services for which the forum
requires pro hac vice admission; or
(2) are services that the lawyer is authorized by federal or other law to provide in this jurisdiction.

XIV) Business Practice


a) Who is a client?
i) Two approaches to representation of entities:
(1) Group theory of representationclient is a collection of individuals who are
represented jointly by the lawyer
(a) Lawyer is required to juggle the conflicts and issues that could arise from
differences among individuals and determine if interests are harmonious
(b) This approach is troubling because you never know when clients may diverge
45

(c) Unworkable approach for large organizations


(d) In small closely held corporations sometimes a lawyer may have obligations
to both because of frequent contact with multiple parties involves.
(i) They may believe attorney represents them personally.
(e) Group theory works well with small informal organizations.
(i) In most cases, the law has opted for the entity theory.
(2) Entity Theoryclearly stated in MR
(a) Lawyer for entity organization not the individuals
(i) Easy rule to state not enforce because entities function through individuals
b) Reporting Up and Reporting Out
i) Once discovered it must be clear that you represent the corporation and not
individuals you routinely deal with.
ii) Question: How does lawyer fulfill obligations to corporation if people you are dealing
with are involved in wrongdoing?
(1) Entity theory prescribes series of responses lawyer should follow (report up the
ladder):
(a) If office/employee is engaged in wrongdoing that is likely to result in
substantial harm, the lawyer shall proceed as is reasonably necessary for the
best interest of organization.
(i) MR 1.13must refer matter up the chain of command to higher authority
in the organization.
(b) Suppose board of directors refuses to act.
(i) MR 1.13 allows lawyer to disclose confidential information to the extent
necessary to avoid substantial injury to the organization (reporting out).
1. Reporting out is PERMISSIVE, not mandatory.
2. Lawyer has to be careful not to over discloserule permits disclosure
MR 1.13Organization as Client
reasonably
necessary
to stop harm
to through
organization.
(a) A lawyer employed or retained byonly
an organization
represents
the organization
acting
its duly authorized constituents.
(b) If a lawyer for an organization knows that an office, employee or other person associated with the organization is engaged in
action, intends to act or refuses to act in a matter related to the representation that is a violation of a legal obligation or the
organization, or a violation of law that reasonably might be imputed to the organization, and that is likely to result in substantial
injury to the organization, then the lawyer shall proceed as is reasonably necessary in the best interest of the organization. Unless
the lawyer reasonably believes that it is not necessary in the best interest of the organization to do so, the lawyer shall refer the
matter to higher authority in the organization, including, if warranted by the circumstances, to the highest authority that can act on
behalf of the organization as determined by applicable law.
(c) Except as provided in Paragraph (d), if,
(1) despite the lawyers efforts in accordance with paragraph (b), the highest authority that can act on behalf of the organization
insists upon or fails to address in a timely and appropriate manner an action, or refusal to act, that is clearly a violation of law, and
(2) the lawyer reasonably believes that the violation is reasonably certain to result in substantial injury to the organization, then
the lawyer may reveal information relating to the representation whether or not Rule 1.6 permits such disclosure, but only if and
to the extent the lawyer reasonably believes necessary t prevent substantial injury to the organization.
(d) Paragraph (c) shall not apply with respect to information relating to a lawyers representation by an organization to investigate
an alleged violation of law, or to defend the organization or an officer, employee or other constituent associated with the
organization against a claim arising out of an alleged violation of law.
(e) A lawyer who reasonably believes that he or she has been discharged because the lawyers actions taken pursuant to Paragraphs
(b) and (c), or who withdraws under circumstances that require or permit the lawyer to take action under either of those paragraphs,
shall proceed as the lawyer reasonably believes necessary to assure that the organizations highest authority is informed of the
lawyers discharge or withdrawal.
(f) In dealing with an organizations directors, officers, employees, members, shareholders or other constituents, a lawyer shall
explain the identity of the client when the lawyer knows or reasonably should know that the organizations interests are adverse to
those of the constituents with whom the lawyer is dealing.
46
(g) A lawyer representing an organization may also represent any of its directors, officers, employees, members, shareholders or
other constituents, subject to the provisions of Rule 1.7. If the organizations consent to the dual representation is required by Rule
1.7, the consent shall be given by an appropriate official of the organization other than the individual who is to be represented, or by
the shareholders.

Summing up Pro Ro:


1) Always tell the truth. Whatever short term advantage you think youll get from lying to
adversary/client/court it is nothing compared to the damage you will do to your career if you are
caught in a lie. Reputation is a very fragile thing. One lie will ruin it all.
2) Play nice. Dont try and play hardball when you dont have to. Be nice and professional.
Dont see every advantage in every situation.
3) Dont misbehave while you live under my roof. Remember you are a lawyer 24/7 wherever
you go and whatever you do. You can be disciplined for things that have nothing to do with your
lawyering. Remember you are held to a higher standard.
4) Dont bite the hand that feeds you. Never take your client for granted. Treat them like human
beings. Have difficult conversations necessary to achieve their objectives, dont assume you
know their objectives. Make sure you are on the same moral/legal page.
5) Be prepared. Know the rules. When you get into practice there are nuances for each
jurisdiction. Recognize the differences by jurisdiction and know them.
6) Alls well that ends well. When it is time to call it quits with a client, follow the rules. Most of
your obligations with a client continue. Duty of confidentiality/loyalty.

47

You might also like